Fall 2025/2026 Update | 100% Correct
QUESTION 1
QUESTION: A 45-year-old male presents with left lower quadrant pain and fever. A CT scan
reveals colonic wall thickening and pericolic fat stranding. What is the most likely diagnosis?
A. Appendicitis
B. Cholecystitis
C. Diverticulitis
D. Ulcerative colitis
CORRECT ANSWER: C
RATIONALE: Diverticulitis typically presents with left lower quadrant pain, fever, and
leukocytosis. CT findings of bowel wall thickening and pericolic fat stranding confirm
inflammation of diverticula. Appendicitis usually causes right lower quadrant pain; cholecystitis
affects the right upper quadrant; ulcerative colitis presents with chronic diarrhea and bloody
stools rather than focal inflammation.
QUESTION 2
QUESTION: A 55-year-old woman presents with exertional chest pain relieved by rest. ECG and
troponin levels are normal. What is the most appropriate initial management?
A. Immediate cardiac catheterization
B. Start sublingual nitroglycerin and prescribe a beta-blocker
C. Begin IV morphine for pain relief
D. Schedule emergency coronary bypass
CORRECT ANSWER: B
RATIONALE: This presentation is consistent with stable angina. Management includes nitrates
for symptomatic relief and beta-blockers to reduce myocardial oxygen demand. Normal
troponins and ECG findings exclude acute coronary syndrome requiring immediate
catheterization.
QUESTION 3
,QUESTION: A patient with COPD presents with increased sputum production, dyspnea, and
purulence. Which of the following therapeutic steps is indicated?
A. Discontinue bronchodilators
B. Initiate systemic corticosteroids and antibiotics
C. Prescribe diuretics only
D. Begin antifungal therapy
CORRECT ANSWER: B
RATIONALE: An acute exacerbation of COPD is characterized by increased sputum purulence and
quantity with dyspnea. Treatment includes systemic corticosteroids and antibiotics to reduce
inflammation and treat bacterial infection. Bronchodilators should be continued, not
discontinued.
QUESTION 4
QUESTION: Which of the following is the first-line treatment for hypertension in the general
non-black adult population according to current guidelines?
A. Beta-blockers
B. ACE inhibitors
C. Loop diuretics
D. Alpha blockers
CORRECT ANSWER: B
RATIONALE: Guidelines recommend ACE inhibitors or ARBs, calcium channel blockers, or
thiazide-type diuretics as initial therapy. Beta-blockers and loop diuretics are not first-line agents
unless other specific comorbidities exist.
QUESTION 5
QUESTION: A 24-year-old female presents with dysuria and urinary frequency. A urinalysis
reveals positive leukocyte esterase and nitrites. What is the most likely diagnosis?
A. Pyelonephritis
B. Interstitial cystitis
C. Acute cystitis
D. Urethritis due to Chlamydia
CORRECT ANSWER: C
,RATIONALE: Acute cystitis (bladder infection) commonly presents with dysuria and frequency
and positive nitrites on urinalysis, suggesting Enterobacteriaceae infection such as E. coli.
Pyelonephritis would show systemic symptoms like fever and flank pain; Chlamydial urethritis
usually lacks nitrite positivity.
QUESTION 6
QUESTION: A patient with a history of alcohol use disorder presents with confusion,
ophthalmoplegia, and ataxia. What is the most likely diagnosis?
A. Korsakoff psychosis
B. Delirium tremens
C. Wernicke encephalopathy
D. Hepatic encephalopathy
CORRECT ANSWER: C
RATIONALE: The triad of confusion, ataxia, and ophthalmoplegia is characteristic of Wernicke
encephalopathy, resulting from thiamine (vitamin B1) deficiency. It is a medical emergency
requiring immediate thiamine administration before glucose to prevent irreversible damage.
QUESTION 7
QUESTION: A 32-year-old man presents with sudden onset of pleuritic chest pain and dyspnea.
He has a history of asthma but is otherwise healthy. A chest X-ray shows a small right apical
pleural line. What is the likely diagnosis?
A. Pulmonary embolism
B. Pneumothorax
C. Pneumonia
D. Pleural effusion
CORRECT ANSWER: B
RATIONALE: A visible pleural line with absent lung markings beyond it indicates pneumothorax.
The acute pleuritic pain and dyspnea further support this diagnosis. Pulmonary embolism would
not typically show a pleural line; pneumonia shows infiltrates instead.
, QUESTION 8
QUESTION: A 60-year-old man with long-standing hypertension presents with tearing chest pain
radiating to the back. Blood pressure is markedly different between arms. What is the most
likely diagnosis?
A. Acute coronary syndrome
B. Aortic dissection
C. Pulmonary embolism
D. Pericarditis
CORRECT ANSWER: B
RATIONALE: Aortic dissection presents with tearing or ripping chest pain and blood pressure
discrepancies between extremities. The mechanism involves an intimal tear, creating a false
lumen within the aortic wall.
QUESTION 9
QUESTION: Which of the following organisms is the most common cause of community-
acquired pneumonia?
A. Streptococcus pneumoniae
B. Haemophilus influenzae
C. Mycoplasma pneumoniae
D. Legionella pneumophila
CORRECT ANSWER: A
RATIONALE: Streptococcus pneumoniae is the leading cause of community-acquired pneumonia,
classically presenting with sudden onset of fever, productive cough, and lobar consolidation on
imaging. Mycoplasma and Legionella are more often atypical causes.
QUESTION 10
QUESTION: A 40-year-old man presents with polyuria, polydipsia, and unintended weight loss.
Laboratory results show fasting glucose 280 mg/dL and HbA1c 9.2%. What is the first step in
management?
A. Begin lifestyle modification and metformin
B. Start insulin therapy immediately